Show existence of $sigma$












1












$begingroup$


Let $K$ be a normal extension of $F$ and $fin F[x]$ be irreducible over $F$.



Let $g_1, g_2$ be irreducible factors of $f$ in the ring $K[x]$.



How could we show that there exists $sigma in G(K/F)$ such that $g_2=sigma (g_1)$ ?



Could you give me a hint for that?










share|cite|improve this question









$endgroup$












  • $begingroup$
    IMHO the same question has appeared here. No answers, though. I'll keep looking...
    $endgroup$
    – Jyrki Lahtonen
    Oct 30 '18 at 6:39










  • $begingroup$
    A more promising match. Further checking required to decide whether this is a dupe. May be this?
    $endgroup$
    – Jyrki Lahtonen
    Oct 30 '18 at 6:44
















1












$begingroup$


Let $K$ be a normal extension of $F$ and $fin F[x]$ be irreducible over $F$.



Let $g_1, g_2$ be irreducible factors of $f$ in the ring $K[x]$.



How could we show that there exists $sigma in G(K/F)$ such that $g_2=sigma (g_1)$ ?



Could you give me a hint for that?










share|cite|improve this question









$endgroup$












  • $begingroup$
    IMHO the same question has appeared here. No answers, though. I'll keep looking...
    $endgroup$
    – Jyrki Lahtonen
    Oct 30 '18 at 6:39










  • $begingroup$
    A more promising match. Further checking required to decide whether this is a dupe. May be this?
    $endgroup$
    – Jyrki Lahtonen
    Oct 30 '18 at 6:44














1












1








1


0



$begingroup$


Let $K$ be a normal extension of $F$ and $fin F[x]$ be irreducible over $F$.



Let $g_1, g_2$ be irreducible factors of $f$ in the ring $K[x]$.



How could we show that there exists $sigma in G(K/F)$ such that $g_2=sigma (g_1)$ ?



Could you give me a hint for that?










share|cite|improve this question









$endgroup$




Let $K$ be a normal extension of $F$ and $fin F[x]$ be irreducible over $F$.



Let $g_1, g_2$ be irreducible factors of $f$ in the ring $K[x]$.



How could we show that there exists $sigma in G(K/F)$ such that $g_2=sigma (g_1)$ ?



Could you give me a hint for that?







abstract-algebra galois-theory






share|cite|improve this question













share|cite|improve this question











share|cite|improve this question




share|cite|improve this question










asked Oct 29 '18 at 19:42









Mary StarMary Star

3,02582268




3,02582268












  • $begingroup$
    IMHO the same question has appeared here. No answers, though. I'll keep looking...
    $endgroup$
    – Jyrki Lahtonen
    Oct 30 '18 at 6:39










  • $begingroup$
    A more promising match. Further checking required to decide whether this is a dupe. May be this?
    $endgroup$
    – Jyrki Lahtonen
    Oct 30 '18 at 6:44


















  • $begingroup$
    IMHO the same question has appeared here. No answers, though. I'll keep looking...
    $endgroup$
    – Jyrki Lahtonen
    Oct 30 '18 at 6:39










  • $begingroup$
    A more promising match. Further checking required to decide whether this is a dupe. May be this?
    $endgroup$
    – Jyrki Lahtonen
    Oct 30 '18 at 6:44
















$begingroup$
IMHO the same question has appeared here. No answers, though. I'll keep looking...
$endgroup$
– Jyrki Lahtonen
Oct 30 '18 at 6:39




$begingroup$
IMHO the same question has appeared here. No answers, though. I'll keep looking...
$endgroup$
– Jyrki Lahtonen
Oct 30 '18 at 6:39












$begingroup$
A more promising match. Further checking required to decide whether this is a dupe. May be this?
$endgroup$
– Jyrki Lahtonen
Oct 30 '18 at 6:44




$begingroup$
A more promising match. Further checking required to decide whether this is a dupe. May be this?
$endgroup$
– Jyrki Lahtonen
Oct 30 '18 at 6:44










1 Answer
1






active

oldest

votes


















0












$begingroup$

Let $L/K$ be the splitting field of $f$. Let $alpha_1$ be a root of $g_1$ and $alpha_2$ be a root of $g_2$. From field theory, since $alpha_1$ and $alpha_2$ have the same minimal polynomial over $F$ and $L/F$ is normal, we get $sigma in operatorname{Aut}(L/F)$ such that $sigma(alpha_1) = alpha_2$. Since $K/F$ is normal, $sigma|_K in operatorname{Aut}(K/F)$, still with $sigma|_K(alpha_1) = alpha_2$.



Now $g_2$ is irreducible in $K$ and $g_2(alpha_2) = 0 in L$, so $g_2$ is the minimal poolynomial of $alpha_2$ over $K$. However, $sigma|_K(g_1)$ is also irreducible in $K$, and $(sigma|_K(g_1))(alpha_2) = (sigma|_K(g_1))(sigma(alpha_1)) = sigma|_K(g_1(alpha_1)) = 0$, so $sigma|_K(g_1)$ is also the minimal polynomial of $alpha_2$ over $K$.



Since minimal polynomial is unique, we conclude $sigma|_K(g_1) = g_2$.





A counter-example when $K/F$ is not normal is $F=Bbb Q$ and $K=Bbb Q(sqrt[3]2)$ and $f = X^3-2 in F[X]$, with $f = (X-sqrt[3]2)(X^2+sqrt[3]2X+sqrt[3]4) in K[X]$.






share|cite|improve this answer









$endgroup$













    Your Answer





    StackExchange.ifUsing("editor", function () {
    return StackExchange.using("mathjaxEditing", function () {
    StackExchange.MarkdownEditor.creationCallbacks.add(function (editor, postfix) {
    StackExchange.mathjaxEditing.prepareWmdForMathJax(editor, postfix, [["$", "$"], ["\\(","\\)"]]);
    });
    });
    }, "mathjax-editing");

    StackExchange.ready(function() {
    var channelOptions = {
    tags: "".split(" "),
    id: "69"
    };
    initTagRenderer("".split(" "), "".split(" "), channelOptions);

    StackExchange.using("externalEditor", function() {
    // Have to fire editor after snippets, if snippets enabled
    if (StackExchange.settings.snippets.snippetsEnabled) {
    StackExchange.using("snippets", function() {
    createEditor();
    });
    }
    else {
    createEditor();
    }
    });

    function createEditor() {
    StackExchange.prepareEditor({
    heartbeatType: 'answer',
    autoActivateHeartbeat: false,
    convertImagesToLinks: true,
    noModals: true,
    showLowRepImageUploadWarning: true,
    reputationToPostImages: 10,
    bindNavPrevention: true,
    postfix: "",
    imageUploader: {
    brandingHtml: "Powered by u003ca class="icon-imgur-white" href="https://imgur.com/"u003eu003c/au003e",
    contentPolicyHtml: "User contributions licensed under u003ca href="https://creativecommons.org/licenses/by-sa/3.0/"u003ecc by-sa 3.0 with attribution requiredu003c/au003e u003ca href="https://stackoverflow.com/legal/content-policy"u003e(content policy)u003c/au003e",
    allowUrls: true
    },
    noCode: true, onDemand: true,
    discardSelector: ".discard-answer"
    ,immediatelyShowMarkdownHelp:true
    });


    }
    });














    draft saved

    draft discarded


















    StackExchange.ready(
    function () {
    StackExchange.openid.initPostLogin('.new-post-login', 'https%3a%2f%2fmath.stackexchange.com%2fquestions%2f2976583%2fshow-existence-of-sigma%23new-answer', 'question_page');
    }
    );

    Post as a guest















    Required, but never shown

























    1 Answer
    1






    active

    oldest

    votes








    1 Answer
    1






    active

    oldest

    votes









    active

    oldest

    votes






    active

    oldest

    votes









    0












    $begingroup$

    Let $L/K$ be the splitting field of $f$. Let $alpha_1$ be a root of $g_1$ and $alpha_2$ be a root of $g_2$. From field theory, since $alpha_1$ and $alpha_2$ have the same minimal polynomial over $F$ and $L/F$ is normal, we get $sigma in operatorname{Aut}(L/F)$ such that $sigma(alpha_1) = alpha_2$. Since $K/F$ is normal, $sigma|_K in operatorname{Aut}(K/F)$, still with $sigma|_K(alpha_1) = alpha_2$.



    Now $g_2$ is irreducible in $K$ and $g_2(alpha_2) = 0 in L$, so $g_2$ is the minimal poolynomial of $alpha_2$ over $K$. However, $sigma|_K(g_1)$ is also irreducible in $K$, and $(sigma|_K(g_1))(alpha_2) = (sigma|_K(g_1))(sigma(alpha_1)) = sigma|_K(g_1(alpha_1)) = 0$, so $sigma|_K(g_1)$ is also the minimal polynomial of $alpha_2$ over $K$.



    Since minimal polynomial is unique, we conclude $sigma|_K(g_1) = g_2$.





    A counter-example when $K/F$ is not normal is $F=Bbb Q$ and $K=Bbb Q(sqrt[3]2)$ and $f = X^3-2 in F[X]$, with $f = (X-sqrt[3]2)(X^2+sqrt[3]2X+sqrt[3]4) in K[X]$.






    share|cite|improve this answer









    $endgroup$


















      0












      $begingroup$

      Let $L/K$ be the splitting field of $f$. Let $alpha_1$ be a root of $g_1$ and $alpha_2$ be a root of $g_2$. From field theory, since $alpha_1$ and $alpha_2$ have the same minimal polynomial over $F$ and $L/F$ is normal, we get $sigma in operatorname{Aut}(L/F)$ such that $sigma(alpha_1) = alpha_2$. Since $K/F$ is normal, $sigma|_K in operatorname{Aut}(K/F)$, still with $sigma|_K(alpha_1) = alpha_2$.



      Now $g_2$ is irreducible in $K$ and $g_2(alpha_2) = 0 in L$, so $g_2$ is the minimal poolynomial of $alpha_2$ over $K$. However, $sigma|_K(g_1)$ is also irreducible in $K$, and $(sigma|_K(g_1))(alpha_2) = (sigma|_K(g_1))(sigma(alpha_1)) = sigma|_K(g_1(alpha_1)) = 0$, so $sigma|_K(g_1)$ is also the minimal polynomial of $alpha_2$ over $K$.



      Since minimal polynomial is unique, we conclude $sigma|_K(g_1) = g_2$.





      A counter-example when $K/F$ is not normal is $F=Bbb Q$ and $K=Bbb Q(sqrt[3]2)$ and $f = X^3-2 in F[X]$, with $f = (X-sqrt[3]2)(X^2+sqrt[3]2X+sqrt[3]4) in K[X]$.






      share|cite|improve this answer









      $endgroup$
















        0












        0








        0





        $begingroup$

        Let $L/K$ be the splitting field of $f$. Let $alpha_1$ be a root of $g_1$ and $alpha_2$ be a root of $g_2$. From field theory, since $alpha_1$ and $alpha_2$ have the same minimal polynomial over $F$ and $L/F$ is normal, we get $sigma in operatorname{Aut}(L/F)$ such that $sigma(alpha_1) = alpha_2$. Since $K/F$ is normal, $sigma|_K in operatorname{Aut}(K/F)$, still with $sigma|_K(alpha_1) = alpha_2$.



        Now $g_2$ is irreducible in $K$ and $g_2(alpha_2) = 0 in L$, so $g_2$ is the minimal poolynomial of $alpha_2$ over $K$. However, $sigma|_K(g_1)$ is also irreducible in $K$, and $(sigma|_K(g_1))(alpha_2) = (sigma|_K(g_1))(sigma(alpha_1)) = sigma|_K(g_1(alpha_1)) = 0$, so $sigma|_K(g_1)$ is also the minimal polynomial of $alpha_2$ over $K$.



        Since minimal polynomial is unique, we conclude $sigma|_K(g_1) = g_2$.





        A counter-example when $K/F$ is not normal is $F=Bbb Q$ and $K=Bbb Q(sqrt[3]2)$ and $f = X^3-2 in F[X]$, with $f = (X-sqrt[3]2)(X^2+sqrt[3]2X+sqrt[3]4) in K[X]$.






        share|cite|improve this answer









        $endgroup$



        Let $L/K$ be the splitting field of $f$. Let $alpha_1$ be a root of $g_1$ and $alpha_2$ be a root of $g_2$. From field theory, since $alpha_1$ and $alpha_2$ have the same minimal polynomial over $F$ and $L/F$ is normal, we get $sigma in operatorname{Aut}(L/F)$ such that $sigma(alpha_1) = alpha_2$. Since $K/F$ is normal, $sigma|_K in operatorname{Aut}(K/F)$, still with $sigma|_K(alpha_1) = alpha_2$.



        Now $g_2$ is irreducible in $K$ and $g_2(alpha_2) = 0 in L$, so $g_2$ is the minimal poolynomial of $alpha_2$ over $K$. However, $sigma|_K(g_1)$ is also irreducible in $K$, and $(sigma|_K(g_1))(alpha_2) = (sigma|_K(g_1))(sigma(alpha_1)) = sigma|_K(g_1(alpha_1)) = 0$, so $sigma|_K(g_1)$ is also the minimal polynomial of $alpha_2$ over $K$.



        Since minimal polynomial is unique, we conclude $sigma|_K(g_1) = g_2$.





        A counter-example when $K/F$ is not normal is $F=Bbb Q$ and $K=Bbb Q(sqrt[3]2)$ and $f = X^3-2 in F[X]$, with $f = (X-sqrt[3]2)(X^2+sqrt[3]2X+sqrt[3]4) in K[X]$.







        share|cite|improve this answer












        share|cite|improve this answer



        share|cite|improve this answer










        answered Jan 1 at 23:53









        Kenny LauKenny Lau

        19.9k2159




        19.9k2159






























            draft saved

            draft discarded




















































            Thanks for contributing an answer to Mathematics Stack Exchange!


            • Please be sure to answer the question. Provide details and share your research!

            But avoid



            • Asking for help, clarification, or responding to other answers.

            • Making statements based on opinion; back them up with references or personal experience.


            Use MathJax to format equations. MathJax reference.


            To learn more, see our tips on writing great answers.




            draft saved


            draft discarded














            StackExchange.ready(
            function () {
            StackExchange.openid.initPostLogin('.new-post-login', 'https%3a%2f%2fmath.stackexchange.com%2fquestions%2f2976583%2fshow-existence-of-sigma%23new-answer', 'question_page');
            }
            );

            Post as a guest















            Required, but never shown





















































            Required, but never shown














            Required, but never shown












            Required, but never shown







            Required, but never shown

































            Required, but never shown














            Required, but never shown












            Required, but never shown







            Required, but never shown







            Popular posts from this blog

            Human spaceflight

            Can not write log (Is /dev/pts mounted?) - openpty in Ubuntu-on-Windows?

            File:DeusFollowingSea.jpg